Integral Closure Flat












1












$begingroup$


Let $R$ be an integral domain and $A$ it's integral closure. Futhermore $A$ is a finitely generated $R$-module.



I want to show that if $A$ is flat over $R$ then already $R = A$ holds.



My attempts: I tried (without success) following: Assume $R neq A$. Let $a in A backslash R$. Since $A$ integral closure there exist a minimal $n$ with



(*) $$a^n +r_{n-1}a^{n-1} +... +r_0 =0$$



Since $R$ integral then $R subset Frac(R)$ and the multiplication map $m: R to A, r mapsto ar$ is injective Since $A$ flat it should be stay injective after tensoring with $-otimes A$.



Now I can multipy $a$ also with all elements from $A$. My hope was to deduce the contradiction using (*) by multiplying with a appropiate polynomial in $a$. The problem is the "constant" term r_0.



I think that I'm very close with this proof to the solution but I don't see the final step. Can anybody help. BTW: Up to now I don't used the finitely generated property.



Source: Liu's "Algebraic Geometry". Futhermore: Could the statement be true without the finiteness assumtion. Liu's says yes but how to show?










share|cite|improve this question









$endgroup$












  • $begingroup$
    See math.stackexchange.com/q/482908/660
    $endgroup$
    – Pierre-Yves Gaillard
    Dec 6 '18 at 14:09












  • $begingroup$
    and math.stackexchange.com/questions/385364/…
    $endgroup$
    – Badam Baplan
    Dec 6 '18 at 23:44










  • $begingroup$
    @BadamBaplan: Hi. Thank you for the reference. You refering to Hagen Knaf's answer, don't you? The only not satisfying point there is that for localizations wrt a prime ideal $mathfrak p$ he assumes that there exist always a $x_{mathfrak p}in B_{mathfrak p}$ with $B_{mathfrak p}=x_{mathfrak p}A_{mathfrak p}$. Could you explain why this is true?
    $endgroup$
    – KarlPeter
    Dec 7 '18 at 0:06












  • $begingroup$
    Actually I much prefer the other answer there, which does not use the assumption that the integral closure is an f.g. $R$-module.
    $endgroup$
    – Badam Baplan
    Dec 7 '18 at 1:11










  • $begingroup$
    The answer you mention is using the fact that f.g. flat modules are locally cyclic. My feeling is that that is not the right answer to this question, since it has nothing to do with the integral closure. That approach shows that a domain has no proper f.g. flat overrings, but the second approach shows that a domain has no proper integral flat overrings. The second result implies the first.
    $endgroup$
    – Badam Baplan
    Dec 7 '18 at 1:21
















1












$begingroup$


Let $R$ be an integral domain and $A$ it's integral closure. Futhermore $A$ is a finitely generated $R$-module.



I want to show that if $A$ is flat over $R$ then already $R = A$ holds.



My attempts: I tried (without success) following: Assume $R neq A$. Let $a in A backslash R$. Since $A$ integral closure there exist a minimal $n$ with



(*) $$a^n +r_{n-1}a^{n-1} +... +r_0 =0$$



Since $R$ integral then $R subset Frac(R)$ and the multiplication map $m: R to A, r mapsto ar$ is injective Since $A$ flat it should be stay injective after tensoring with $-otimes A$.



Now I can multipy $a$ also with all elements from $A$. My hope was to deduce the contradiction using (*) by multiplying with a appropiate polynomial in $a$. The problem is the "constant" term r_0.



I think that I'm very close with this proof to the solution but I don't see the final step. Can anybody help. BTW: Up to now I don't used the finitely generated property.



Source: Liu's "Algebraic Geometry". Futhermore: Could the statement be true without the finiteness assumtion. Liu's says yes but how to show?










share|cite|improve this question









$endgroup$












  • $begingroup$
    See math.stackexchange.com/q/482908/660
    $endgroup$
    – Pierre-Yves Gaillard
    Dec 6 '18 at 14:09












  • $begingroup$
    and math.stackexchange.com/questions/385364/…
    $endgroup$
    – Badam Baplan
    Dec 6 '18 at 23:44










  • $begingroup$
    @BadamBaplan: Hi. Thank you for the reference. You refering to Hagen Knaf's answer, don't you? The only not satisfying point there is that for localizations wrt a prime ideal $mathfrak p$ he assumes that there exist always a $x_{mathfrak p}in B_{mathfrak p}$ with $B_{mathfrak p}=x_{mathfrak p}A_{mathfrak p}$. Could you explain why this is true?
    $endgroup$
    – KarlPeter
    Dec 7 '18 at 0:06












  • $begingroup$
    Actually I much prefer the other answer there, which does not use the assumption that the integral closure is an f.g. $R$-module.
    $endgroup$
    – Badam Baplan
    Dec 7 '18 at 1:11










  • $begingroup$
    The answer you mention is using the fact that f.g. flat modules are locally cyclic. My feeling is that that is not the right answer to this question, since it has nothing to do with the integral closure. That approach shows that a domain has no proper f.g. flat overrings, but the second approach shows that a domain has no proper integral flat overrings. The second result implies the first.
    $endgroup$
    – Badam Baplan
    Dec 7 '18 at 1:21














1












1








1





$begingroup$


Let $R$ be an integral domain and $A$ it's integral closure. Futhermore $A$ is a finitely generated $R$-module.



I want to show that if $A$ is flat over $R$ then already $R = A$ holds.



My attempts: I tried (without success) following: Assume $R neq A$. Let $a in A backslash R$. Since $A$ integral closure there exist a minimal $n$ with



(*) $$a^n +r_{n-1}a^{n-1} +... +r_0 =0$$



Since $R$ integral then $R subset Frac(R)$ and the multiplication map $m: R to A, r mapsto ar$ is injective Since $A$ flat it should be stay injective after tensoring with $-otimes A$.



Now I can multipy $a$ also with all elements from $A$. My hope was to deduce the contradiction using (*) by multiplying with a appropiate polynomial in $a$. The problem is the "constant" term r_0.



I think that I'm very close with this proof to the solution but I don't see the final step. Can anybody help. BTW: Up to now I don't used the finitely generated property.



Source: Liu's "Algebraic Geometry". Futhermore: Could the statement be true without the finiteness assumtion. Liu's says yes but how to show?










share|cite|improve this question









$endgroup$




Let $R$ be an integral domain and $A$ it's integral closure. Futhermore $A$ is a finitely generated $R$-module.



I want to show that if $A$ is flat over $R$ then already $R = A$ holds.



My attempts: I tried (without success) following: Assume $R neq A$. Let $a in A backslash R$. Since $A$ integral closure there exist a minimal $n$ with



(*) $$a^n +r_{n-1}a^{n-1} +... +r_0 =0$$



Since $R$ integral then $R subset Frac(R)$ and the multiplication map $m: R to A, r mapsto ar$ is injective Since $A$ flat it should be stay injective after tensoring with $-otimes A$.



Now I can multipy $a$ also with all elements from $A$. My hope was to deduce the contradiction using (*) by multiplying with a appropiate polynomial in $a$. The problem is the "constant" term r_0.



I think that I'm very close with this proof to the solution but I don't see the final step. Can anybody help. BTW: Up to now I don't used the finitely generated property.



Source: Liu's "Algebraic Geometry". Futhermore: Could the statement be true without the finiteness assumtion. Liu's says yes but how to show?







ring-theory commutative-algebra flatness






share|cite|improve this question













share|cite|improve this question











share|cite|improve this question




share|cite|improve this question










asked Dec 6 '18 at 0:40









KarlPeterKarlPeter

6081315




6081315












  • $begingroup$
    See math.stackexchange.com/q/482908/660
    $endgroup$
    – Pierre-Yves Gaillard
    Dec 6 '18 at 14:09












  • $begingroup$
    and math.stackexchange.com/questions/385364/…
    $endgroup$
    – Badam Baplan
    Dec 6 '18 at 23:44










  • $begingroup$
    @BadamBaplan: Hi. Thank you for the reference. You refering to Hagen Knaf's answer, don't you? The only not satisfying point there is that for localizations wrt a prime ideal $mathfrak p$ he assumes that there exist always a $x_{mathfrak p}in B_{mathfrak p}$ with $B_{mathfrak p}=x_{mathfrak p}A_{mathfrak p}$. Could you explain why this is true?
    $endgroup$
    – KarlPeter
    Dec 7 '18 at 0:06












  • $begingroup$
    Actually I much prefer the other answer there, which does not use the assumption that the integral closure is an f.g. $R$-module.
    $endgroup$
    – Badam Baplan
    Dec 7 '18 at 1:11










  • $begingroup$
    The answer you mention is using the fact that f.g. flat modules are locally cyclic. My feeling is that that is not the right answer to this question, since it has nothing to do with the integral closure. That approach shows that a domain has no proper f.g. flat overrings, but the second approach shows that a domain has no proper integral flat overrings. The second result implies the first.
    $endgroup$
    – Badam Baplan
    Dec 7 '18 at 1:21


















  • $begingroup$
    See math.stackexchange.com/q/482908/660
    $endgroup$
    – Pierre-Yves Gaillard
    Dec 6 '18 at 14:09












  • $begingroup$
    and math.stackexchange.com/questions/385364/…
    $endgroup$
    – Badam Baplan
    Dec 6 '18 at 23:44










  • $begingroup$
    @BadamBaplan: Hi. Thank you for the reference. You refering to Hagen Knaf's answer, don't you? The only not satisfying point there is that for localizations wrt a prime ideal $mathfrak p$ he assumes that there exist always a $x_{mathfrak p}in B_{mathfrak p}$ with $B_{mathfrak p}=x_{mathfrak p}A_{mathfrak p}$. Could you explain why this is true?
    $endgroup$
    – KarlPeter
    Dec 7 '18 at 0:06












  • $begingroup$
    Actually I much prefer the other answer there, which does not use the assumption that the integral closure is an f.g. $R$-module.
    $endgroup$
    – Badam Baplan
    Dec 7 '18 at 1:11










  • $begingroup$
    The answer you mention is using the fact that f.g. flat modules are locally cyclic. My feeling is that that is not the right answer to this question, since it has nothing to do with the integral closure. That approach shows that a domain has no proper f.g. flat overrings, but the second approach shows that a domain has no proper integral flat overrings. The second result implies the first.
    $endgroup$
    – Badam Baplan
    Dec 7 '18 at 1:21
















$begingroup$
See math.stackexchange.com/q/482908/660
$endgroup$
– Pierre-Yves Gaillard
Dec 6 '18 at 14:09






$begingroup$
See math.stackexchange.com/q/482908/660
$endgroup$
– Pierre-Yves Gaillard
Dec 6 '18 at 14:09














$begingroup$
and math.stackexchange.com/questions/385364/…
$endgroup$
– Badam Baplan
Dec 6 '18 at 23:44




$begingroup$
and math.stackexchange.com/questions/385364/…
$endgroup$
– Badam Baplan
Dec 6 '18 at 23:44












$begingroup$
@BadamBaplan: Hi. Thank you for the reference. You refering to Hagen Knaf's answer, don't you? The only not satisfying point there is that for localizations wrt a prime ideal $mathfrak p$ he assumes that there exist always a $x_{mathfrak p}in B_{mathfrak p}$ with $B_{mathfrak p}=x_{mathfrak p}A_{mathfrak p}$. Could you explain why this is true?
$endgroup$
– KarlPeter
Dec 7 '18 at 0:06






$begingroup$
@BadamBaplan: Hi. Thank you for the reference. You refering to Hagen Knaf's answer, don't you? The only not satisfying point there is that for localizations wrt a prime ideal $mathfrak p$ he assumes that there exist always a $x_{mathfrak p}in B_{mathfrak p}$ with $B_{mathfrak p}=x_{mathfrak p}A_{mathfrak p}$. Could you explain why this is true?
$endgroup$
– KarlPeter
Dec 7 '18 at 0:06














$begingroup$
Actually I much prefer the other answer there, which does not use the assumption that the integral closure is an f.g. $R$-module.
$endgroup$
– Badam Baplan
Dec 7 '18 at 1:11




$begingroup$
Actually I much prefer the other answer there, which does not use the assumption that the integral closure is an f.g. $R$-module.
$endgroup$
– Badam Baplan
Dec 7 '18 at 1:11












$begingroup$
The answer you mention is using the fact that f.g. flat modules are locally cyclic. My feeling is that that is not the right answer to this question, since it has nothing to do with the integral closure. That approach shows that a domain has no proper f.g. flat overrings, but the second approach shows that a domain has no proper integral flat overrings. The second result implies the first.
$endgroup$
– Badam Baplan
Dec 7 '18 at 1:21




$begingroup$
The answer you mention is using the fact that f.g. flat modules are locally cyclic. My feeling is that that is not the right answer to this question, since it has nothing to do with the integral closure. That approach shows that a domain has no proper f.g. flat overrings, but the second approach shows that a domain has no proper integral flat overrings. The second result implies the first.
$endgroup$
– Badam Baplan
Dec 7 '18 at 1:21










0






active

oldest

votes











Your Answer





StackExchange.ifUsing("editor", function () {
return StackExchange.using("mathjaxEditing", function () {
StackExchange.MarkdownEditor.creationCallbacks.add(function (editor, postfix) {
StackExchange.mathjaxEditing.prepareWmdForMathJax(editor, postfix, [["$", "$"], ["\\(","\\)"]]);
});
});
}, "mathjax-editing");

StackExchange.ready(function() {
var channelOptions = {
tags: "".split(" "),
id: "69"
};
initTagRenderer("".split(" "), "".split(" "), channelOptions);

StackExchange.using("externalEditor", function() {
// Have to fire editor after snippets, if snippets enabled
if (StackExchange.settings.snippets.snippetsEnabled) {
StackExchange.using("snippets", function() {
createEditor();
});
}
else {
createEditor();
}
});

function createEditor() {
StackExchange.prepareEditor({
heartbeatType: 'answer',
autoActivateHeartbeat: false,
convertImagesToLinks: true,
noModals: true,
showLowRepImageUploadWarning: true,
reputationToPostImages: 10,
bindNavPrevention: true,
postfix: "",
imageUploader: {
brandingHtml: "Powered by u003ca class="icon-imgur-white" href="https://imgur.com/"u003eu003c/au003e",
contentPolicyHtml: "User contributions licensed under u003ca href="https://creativecommons.org/licenses/by-sa/3.0/"u003ecc by-sa 3.0 with attribution requiredu003c/au003e u003ca href="https://stackoverflow.com/legal/content-policy"u003e(content policy)u003c/au003e",
allowUrls: true
},
noCode: true, onDemand: true,
discardSelector: ".discard-answer"
,immediatelyShowMarkdownHelp:true
});


}
});














draft saved

draft discarded


















StackExchange.ready(
function () {
StackExchange.openid.initPostLogin('.new-post-login', 'https%3a%2f%2fmath.stackexchange.com%2fquestions%2f3027879%2fintegral-closure-flat%23new-answer', 'question_page');
}
);

Post as a guest















Required, but never shown

























0






active

oldest

votes








0






active

oldest

votes









active

oldest

votes






active

oldest

votes
















draft saved

draft discarded




















































Thanks for contributing an answer to Mathematics Stack Exchange!


  • Please be sure to answer the question. Provide details and share your research!

But avoid



  • Asking for help, clarification, or responding to other answers.

  • Making statements based on opinion; back them up with references or personal experience.


Use MathJax to format equations. MathJax reference.


To learn more, see our tips on writing great answers.




draft saved


draft discarded














StackExchange.ready(
function () {
StackExchange.openid.initPostLogin('.new-post-login', 'https%3a%2f%2fmath.stackexchange.com%2fquestions%2f3027879%2fintegral-closure-flat%23new-answer', 'question_page');
}
);

Post as a guest















Required, but never shown





















































Required, but never shown














Required, but never shown












Required, but never shown







Required, but never shown

































Required, but never shown














Required, but never shown












Required, but never shown







Required, but never shown







Popular posts from this blog

Quarter-circle Tiles

build a pushdown automaton that recognizes the reverse language of a given pushdown automaton?

Mont Emei